Use the order of operations to simplify the following expression.

-2 3 + |7| - 4 · 2

-38
-23
-9
-10

Answers

Answer 1

Answer:

-9

Step-by-step explanation:

-2 3 + |7| - 4 · 2

I assume -2 3 means -2^3.

-2^3 + |7| - 4 · 2 =

= -8 + 7 - 8

= -1 - 8

= -9

Answer 2

Answer:

-9

Step-by-step explanation:

-2^ 3 + |7| - 4 · 2

Parentheses first and an absolute value is considered parentheses

-2 ^3 + 7 - 4 · 2

Then exponenets

Since the sign is outside of the exponent it is considered multiplication

-1 * (2^3)+ 7 - 4 · 2

-1 *8 + 7 - 4 · 2

Then multiply

-8 +7 -8

Then add and subtract from left to right

-1-8

-9


Related Questions

Classify the type of angle.
A: Acute
B: Right
C: Straight
D: Obtuse

Answers

Answer:

Obtuse

Step-by-step explanation:

I really don’t know which angle they’re asking for because the 140 degree angle is obtuse and the 40 degree angle is acute

Answer:

obtuse angle that's the answer

Abu is trying to decide which pet–sitting service he wants to use . Your Pets charges a $15 fee, plus $1 .75 per hour . Sit Pets charges an $11 fee, plus $2 .25 per hour . At how many hours will both services ...

Answers

Answer:I did not see the entire question but is assuming the question is asking how many hours for both services to cost the same.

Your Pets Cost =15+1.75x

Sit Pets Cost =11+2.25x

set both equations equal to each other

15+1.75x=11+2.25x

15-11 = (2.25-1.75)x

4=0.5x

x=8

Step-by-step explanation:

821) The integon which is 15 more than - 55 is

Answers

Answer:

-40

Step-by-step explanation:

-55 + 15 = x

-40 =x

Solve the inequality (help please)

Answers

Answer:

v<1 23/25

Step-by-step explanation:

The inequality simplifies to 48/25, which is equivalent to 1 23/25.

Write the equation of the line in point-slope form given the information:

Slope = -1/5

Y intercept = -3

Answers

point slope form:

y + 3= -1/5 x (x+0)

Step-by-step explanation:

-use the point slope form equation: y - y1 = m (x - x1).

-using the given information we know that m = -1/5 and that the y intercept is (0,-3).

- y and x will be kept as a variable.

-plug in the y1 and x1 from the y intercept:

y + 3 = m(x + 0)

-once you've plugged in the y intercept then plug in the slope which gives you the answer:

y + 3= -1/5 x (x+0)

I need help:/ I’m in college

Answers

Step-by-step explanation:

Amount of acid = 14.9% of 331 mL solution

= 0.149×(331 mL)

= 49.3 mL acid

Blood pressure values are often reported to the nearest 5 mmhg (100, 105, 110, etc.). the actual blood pressure values for nine randomly selected individuals are given below.

108.6 117.4 128.4 120.0 103.7 112.0 98.3 121.5 123.2

Required:
a. What is the median of the reported blood pressure values?
b. Suppose the blood pressure of the second individual is 117.7 rather than 117.4 (a small change in a single value). What is the new median of the reported values?
c. What does this say about the sensitivity of the median to rounding or grouping in the data?

Answers

Answer:

Step-by-step explanation:

Arranging the data in the ascending order:

108.6 98.3 103.7 112 117.4 120 121.5 123.2 128.4

The median is the middle value of the data set:

a)

Hence,

median = 117.4

b)

When the value of blood pressure is 117.7 instead of 117.4 then the median will be:

Median = 117.7

c)

This indicates that the median of a well sorted set of data is depends upon the middle value of the data set.

Answer question below

Answers

Answer:

edge of cube = root 13.5/6 = 1.5

volume of cube = (Edge of cube)^3= (1.5)^3 = 3.37500 m^3

what is the completely factored form of this expression?

y^2-12y+32

a.(y+4)(y+8)
b.(y-4)(y-8)
c.(y+18)(y+2)
d.(y-18)(y-2)

Answers

(y-4)(y-8)
Because
-4-8=-12
-4*-8=32

Answer:

[tex]y^2-12y+32[/tex]

[tex]=(x - 4)(x - 8)[/tex]

------------------------

Hope it helps...

Have a great day!!

PLEASE PLEASE HELP ASAPPPP IM BEING TIMEDDD

6x2y − 3xy − 24xy2 + 12y2
Rewrite the expression completely factored. Show the steps of your work.

Answers

Answer:

3y(2x-1)(x-4y)

Step-by-step explanation:

Apply exponent rule:

6x^2y-3xy-24xyy+12yy

Rewrite 12 as 4*3

Rewrite -24 as 8*3

Rewrite 6 as 2*3

2*3x^2y-3xy+8*3xyy+4*3yy

Factor out common term 3y:

3y(2x^2-x-8xy+4y)

Factor 2x^2-x-8xy+4y:

3y(2x-1)(x-4y)

Your Answer Is 3y(2x-1)(x-4y)

4. Five cards are randomly chosen from a deck of 52 (13 denominations with 4 suits). a. How many ways are there to receive 5 cards from a deck of 52

Answers

Answer:

There are 2,598,960 ways to receive 5 cards from a deck of 52.

Step-by-step explanation:

The order in which the cards are chosen is not important, which means that the combinations formula is used to solve this question.

Combinations formula:

[tex]C_{n,x}[/tex] is the number of different combinations of x objects from a set of n elements, given by the following formula.

[tex]C_{n,x} = \frac{n!}{x!(n-x)!}[/tex]

a. How many ways are there to receive 5 cards from a deck of 52?

[tex]C_{52,5} = \frac{52!}{5!(47)!} = 2598960[/tex]

There are 2,598,960 ways to receive 5 cards from a deck of 52.

The diameter of the base is the cone measured 8 units. The height measures 6 units.

What is the volume of the cone?

A) 24 π cubic units

B) 32 π cubic units

C)48 π cubic units

D)64 π cubic units

Answers

The answer is B) 32 cubic units

What is the range for the following set of numbers?57, -5, 11, 39, 56, 82, -2, 11, 64, 18, 37, 15, 68

Answers

The range= the highest number - the lowest number

so

82-(-2)

=84

then ur answer is 84

a recent survey shows that 16% of college students have dogs and 38% have an HBO subscription. assuming these two events are independent, what is the probability that a randomly selected college student has neither a dog nor HBO

Answers

Answer: [tex]0.939\ or\ 93.9\%[/tex]

Step-by-step explanation:

Given

Survey shows that 16% of college students have dogs and 38% have HBO subscription

Probability that a random person have both is

[tex]\Rightarrow P_o=0.16\times 0.38\quad [\text{As both events are independent}]\\\Rightarrow P_o=0.0608[/tex]

The probability that the random person has neither of the two is

[tex]\Rightarrow P=1-P_o\\\Rightarrow P=1-0.0608\\\Rightarrow P=0.939[/tex]

If 400 patrons visit the park in March and 550 patrons visit in April, the total number of patrons who
visited the park over the two months falls into all of the following categories except
O real numbers
O rational numbers
o irrational numbers

Answers

950 is a real number, and it is a rational number since it can be expressed as 950/1. Therefore, it falls into all of the categories expect irrational numbers.

Solve and check
y − 5 = −5​

Answers

Answer:

Its -0

Step-by-step explanation:

-0-5=-5

Hope thia helps I believe its right check on a calculator

Help me with this please!!

Answers

Answer:

[tex]y=\sqrt{x} -2[/tex]

Step-by-step explanation:

If you were to replace x with 0 in each of the equations, you would get

from top to bottom choices:

[tex]\sqrt{2}[/tex]

[tex]\sqrt{-2}[/tex]

2

-2

Since we can see that the line touches the y-axis on the number -2, we know that it is the last choice that is the answer

Two planes are the same altitude. From the airport , one plane is 50 km away in the direction of N°60 E and another is 80 km away in the direction of S50° E .How far apart are the two planes

Answers

9514 1404 393

Answer:

  78.5 km

Step-by-step explanation:

Measured at the airport, the angle between the two planes is ...

  180° -60° -50° = 70°

The law of cosines tells us the distance between the planes is ...

  d = √(50² +80² -2·50·80·cos(70°)) ≈ √6163.84 ≈ 78.5 . . . km

The planes are about 78.5 km apart.

Matt buys a new fish tank. The fish tank is in the shape of a cuboid. The diagram shows water in the tank. 30 cm 30 cm 100 cm Matt knows 1000 cm' = 1 litre 1 gallons = 4.5 litres He can keep 2 small fish in the tank for every 1 gallon of water in the tank. Matt thinks he can keep more than 36 small fish in the tank. Is Matt correct?​

Answers

Answer: Yes, but only if he houses 37, 38, 39, or 40 fish

Anything larger than 40 and he'll need more room.

==========================================================

Explanation:

The tank is 30 cm by 30 cm by 100 cm. The volume is 30*30*100 = 90,000 cm^3 which is shorthand for "cubic centimeters".

We're told that 1000 cm^3 = 1 liter, which means the 90,000 cm^3 converts to (90,000)/(1000) = 90 liters.

The fish tank is 90 liters.

Since 1 gallon = 4.5 liters, this means the 90 liter tank converts to 90/(4.5) = 20 gallons

----------------------------

Your teacher mentions "He can keep 2 small fish for every 1 gallon".

Since the tank is 20 gallons, that means he can keep 20*2 = 40 fish. This value is larger than 36, so Matt is correct to a point. If Matt is thinking 37, 38, 39, or 40 fish then he would be correct. If Matt is wanting more than 40 fish, then he'll need a bigger tank.

In short, he can't have any number over 36 and can only have 4 specific values (the four values mentioned earlier).

So technically, Matt is correct, but strong clarification is needed.

Given the triangle below, what is m 2A, rounded to the nearest tenth?
B
18
11
309
A
C
Triangle not drawn to scale
A. 17.8
OB. 20.4
O C. 24.2
D. 26.8

Answers

9514 1404 393

Answer:

  A.  17.8°

Step-by-step explanation:

We have an angle and an opposite side, so we can use the law of sines to find the angle of interest.

  sin(A)/a = sin(C)/c

  A = arcsin(a/c·sin(C)) = arcsin(11/18·1/2) = arcsin(11/36)

  A ≈ 17.7916° ≈ 17.8°

28, 45, 12, 34, 36, 45, 19,20
Part 1: Find the mean of this set of data.
Part 2: Find the mean absolute deviation of this set of data.
Part 3: Using complete sentences, explain what your results for part A and B mean.
i really need an answer for part c

Answers

Part A

To find the mean, we add up the values and divide by n = 8 since there are 8 values in this set.

Adding the values gets us

28+45+12+34+36+45+19+20 = 239

Dividing this over 8 then leads to 239/8 = 29.875

Answer: 29.875

============================================================

Part B

We'll subtract each data value from the mean. We apply absolute value to ensure the result is never negative.

|28 - 29.875| = 1.875 |45 - 29.875| = 15.125 |12 - 29.875| = 17.875 |34 - 29.875| = 4.125 |36 - 29.875| = 6.125 |45 - 29.875| = 15.125 |19 - 29.875| = 10.875 |20 - 29.875| = 9.875

The list of results we get so far is:

1.875, 15.125, 17.875, 4.125, 6.125, 15.125, 10.875, 9.875

This represents the distance each value is from the mean.

Add these values up and divide by n = 8

1.875+15.125+17.875+4.125+6.125+15.125+10.875+9.875 = 81

81/8 = 10.125

Answer: 10.125

============================================================

Part C

The result of part A is one way to measure the center of the distribution of values. It's the average value, which can more or less represent the entire group. Think of it being like how people vote in a senator to represent them in congress. Ideally, this senator is a supposed "average" person to represent everyone.

The result of part B builds on what part A found. The result of part B is the average distance each value is from the center. This is because each time we subtracted and applied absolute value, we found the distance that item was from the mean.

Example: The calculation |28 - 29.875| = 1.875 shows that 28 is exactly 1.875 units from the mean 29.875

By adding up those results and dividing by 8, we are finding the average distance from the mean. Effectively, it tells us how spread out the data set is. The mean absolute deviation (MAD) is a measure of spread in a similar fashion that the standard deviation is, or in a more looser sense, the range is as well.

---------------

In short, the result of part A is a measure of center while the result of part B is a measure of spread. I use "a" instead of "the" because there are other measures of center and other measures of spread.

Hello everyone can someone answer this question please

Answers

9514 1404 393

Answer:

  (a)  2

Step-by-step explanation:

Each inch is 2.54 cm, so 5.08 cm is ...

  x / (5.08 cm) = (1 in) / (2.54 cm)

  x = (1 in)(5.08/2.54) = (1 in)(2)

  x = 2 in

5.08 cm equals 2 inches.

Write the equation of each line in slope intercept form. Slope is -6, and (1,-2) is on the line

Answers

Y = -6x + 4

See the attached photo for further reference.

Hope this helps! Please make me the brainliest, it’s not necessary but appreciated, I put a lot of effort and research into my answers. Have a good day, stay safe and stay healthy.

what is the value of x, given that figure MNOP is a trapezoid with median qr A. 12 B. 8 C. 6 D. 16

Answers

The answer is D. Because if we put 16 in place of x, MP, NO will be 24, 16 respectedly

Step-by-step explanation:

So all horizontal line will be decrease from line MP to No.

As result MP will be = 24, QR = 20, NO = 16. We can see there's different of 4.

The value of x in the trapezoid is 16.

Option D is the correct answer.

What is a trapezium?

It is a quadrilateral that has one pair of parallel sides and a height.

The area is calculated as: 1/2 x sum of the parallel sides x height.

Examples:

Area of a trapezium that has the parallel sides as 3 cm and 4 cm and a heght o 5 cm.

Area = 1/2 x (3 + 4) x 5

Area = 1/2 x 7 x 5

Area = 35/2 = 17.5 cm^2

We have,

In a trapezoid, the median is the average of the parallel sides,

So we have:

QR

= (NO + MP)/2

= (x + x + 8)/2

= (2x + 8)/2

= x + 4

Since we also know that QR = 20, we can set the two expressions equal to each other and solve for x:

x + 4 = 20

x = 16

Therefore,

The value of x is 16.

Learn more about trapezium here:

https://brainly.com/question/22607187

#SPJ7

Find the lengths the missing side

Answers

Answer:

Long leg = 10√3Short leg = 10Hypotenuse = 20

Step-by-step explanation:

Concept:

Here, we need to know the idea of a special triangle 30-60-90.

A special right triangle is a right triangle with some regular feature that makes calculations on the triangle easier, or for which simple formulas exist.

The ratio between the corresponding side of each angle is 1 : √3 : 2

If you are still confused, please refer to the attachment below for a graphical explanation.

Solve:

30-60-90 ⇔ 1 : √3 : 2

Given the side corresponding to 90° is 20

30° : 90° = 1 : 2

30° : 20 = 1 : 2

30° = 10

30° : 60° = 1 : √3

10 : 60° = 1 : √3

60° = 10√3

Hope this helps!! :)

Please let me know if you have any questions

Yousef is making cookies for a friend's birthday. The recipe only makes one dozen cookies, and he wants to take 4 dozen to the birthday
party. If he needs 1 cups of flour to make one dozen, how much flour will he need to make 4 dozen? Simplify your answer and write it as
a mixed number

Answers

Answer:

6 and 2/3

Step-by-step explanation:

you do 1 2/3 x 4 because that is the amount of flour you need to make one batch. That is the correct answer.

PLEASE HELP ASAPPPPPP!!!! (answer in decimal)

Answers

Answer:

465/1178 = .395 = 39.5%

Step-by-step explanation:

        224 + 245

224+ 387 + 245 + 322

465/1178 = .395 = 39.5%

One of the angles of a pair of complementary angles is 50°. Find the ratio of the pair of complementary angles​

Answers

Answer:

4:5

Step-by-step explanation:

A pair of complementary add up to 90, given that one of them is 50, we can do this: x + 50 = 90

solve it and we get x = 40

40 + 50 = 90, so 40 and 50 are the pair of complementary angles.

As for it's Ratio, we can just get the pair and have it as 40:50 but we can reduce it to 4:5

what value makes the equation

Answers

Answer:

n = .25

Step-by-step explanation:

For the equation to be true, both sides must be equal.

20 * 4 * 3 * n = 5 * 12

240 * n = 60

n = 60/240

n = .25

Answer:

1/4

Step-by-step explanation:

20×n×4×3=5×12

or, 240n=60

or, n=60/240

or, n=1/4

Answered by GAUTHMATH

Parallelograms geometry acellus pls help

Answers

Answer:

x = 60

Step-by-step explanation:

Recall: one of the properties of a parallelogram is that consecutive angles are supplementary. This means they add up to given 180°

2x and x are consecutive angles. Therefore:

2x + x = 180

3x = 180

Divide both sides by 3

3x/3 = 180/3

x = 60

Other Questions
Rs 600 will gave Rs 90 simple interest at 5 % per annum. Neil conducted a study on a group of women in which they viewed cartoons for an hour under one of two environmental conditions "either alone or with other participants. After the study, the participants rated how much they enjoyed watching the cartoons under the two different conditions. What can be concluded based on the results of this study? why cells are sometimes called the building block of life? Coordinates of C and D are (-14 , 6) and (x , 13) respectively.Given x is a positive integer and the distance CD is 25 units.Find the value of x. what is Employment equity act? A storeowner orders 25 calculators that cost $38 each. The storeowner can sell each calculator for $42. The storeowner sold 22 calculators to customers. He had to return 3 calculators that were never sold and pay a $2 charge for each returned calculator (although the initial cost is refunded). What is the storeowner's profit? A square steel bar has a length of 5.1 ft and a 2.7 in by 2.7 in cross section and is subjected to axial tension. The final length is 5.10295 ft . The final side length is 2.69953 in . What is Poisson's ratio for the material Construct an amortization schedule for the first three months and the final three months of payments for a 30-year, 7 percent mortgage in the amount of $90,000. What percentage of the third payment is principal Who was Theodore Roosevelt and how was he affected by succession? If 5k = -25, then 5k - 1 = -25 - 1Segment proof What is the difference between telling and communication. Is it a misconception to say they are the same Midsouth Stitchery wants to improve their productivity. Their process yield is currently 91.56% based on 9816 yards of material. If they are going to improve their productivity to 92.10, how many yards of finished material will they have to produce from the same amount of material input In the reaction A + B + C + D, what are the reactants?O A. Just BB. Cand DO c. A and BO D. A and C I need help please this got me soo confused how do I simplify when there are fractions on fractions A mayor serving in a major metropolitan area receives an internal memorandum indicating personnel at many police stations are single-race. At the time of the report, thirty percent of the police force was black or Hispanic. She immediately calls a press conference and orders transfers of police officers to achieve racial balance across the city. The transferred police officers sue on constitutional grounds. Assuming just these facts, what is the strongest argument that might be advanced by the transferred officers based on constitutional grounds What are the 6 rules in subject verb agreement The area between z=.34 and z=1.93 Select the correct answer.Who can propose an amendment to the Constitution?A a simple majority of CongressB. the president.the Supreme CourtD.a two-thirds majority of Congress PLS HELP :) BRAINLIEST!! Choose two people and explain why the persons you chose were important in Americas fight for independence.